Mathcenter Forum  

Go Back   Mathcenter Forum > คณิตศาสตร์ทั่วไป > ปัญหาคณิตศาสตร์ทั่วไป
สมัครสมาชิก คู่มือการใช้ รายชื่อสมาชิก ปฏิทิน ข้อความวันนี้

ตั้งหัวข้อใหม่ Reply
 
เครื่องมือของหัวข้อ ค้นหาในหัวข้อนี้
  #1  
Old 26 กันยายน 2016, 13:25
อัจฉริยะ อัจฉริยะ ไม่อยู่ในระบบ
กระบี่ไว
 
วันที่สมัครสมาชิก: 09 มิถุนายน 2016
ข้อความ: 217
อัจฉริยะ is on a distinguished road
Default The shortest proof in the world of the Fermat's last theorem

ผมเอามาให้ดูกันแล้วครับ ผิดถูกยังไง ก็แนะนำกันด้วย

ขอบอกว่า ผมใช้เวลาในการเขียน paper แค่ครึ่งชั่วโมง อาจจะตกๆหล่นๆ ในเรื่องไวยากรณ์ภาษาอังกฤษ แต่คิดว่าคงจะอ่านกันรู้เรื่อง

รีบๆเขียนออกมา เห็นว่าเรียกร้องอยากดูกัน อ่านแล้วห้วนๆนะ ผมขี้เกียจพิมพ์เยอะ เสียเวลา ถ้าติดตรงไหนถามได้ครับ

ที่จริงผมข้ามการพิสูจน์บางอย่างไป แต่ก็ละเอาไว้ในฐานที่เข้าใจ




ทีแรกผมใช้วิธีเขียนกราฟ แล้วมันแก้ไม่ได้ เลยท้อใจ คิดไม่ออก เลยลองๆคิดแบบนี้ดู ใช้เวลาคิดแค่ 1 วันครับ แต่ดองไว้นาน เพราะงานยุ่งไม่ค่อยมีเวลาเขียนจริงๆจังๆ
__________________
วิศวกรรมศาสตร์ พระจอมเกล้าลาดกระบัง รุ่น 38

22 เมษายน 2024 18:50 : ข้อความนี้ถูกแก้ไขแล้ว 4 ครั้ง, ครั้งล่าสุดโดยคุณ อัจฉริยะ
ตอบพร้อมอ้างอิงข้อความนี้
  #2  
Old 26 กันยายน 2016, 13:37
อัจฉริยะ อัจฉริยะ ไม่อยู่ในระบบ
กระบี่ไว
 
วันที่สมัครสมาชิก: 09 มิถุนายน 2016
ข้อความ: 217
อัจฉริยะ is on a distinguished road
Default

มีใครพิสูจน์ได้สั้นกว่านี้อีกมั้ยครับ บอกแล้วเอาจริงๆ 3-4 บรรทัดก็จบ

ต่อไปจะตีพิมพ์ Goldbach'conjecture ครับ
__________________
วิศวกรรมศาสตร์ พระจอมเกล้าลาดกระบัง รุ่น 38

26 กันยายน 2016 13:38 : ข้อความนี้ถูกแก้ไขแล้ว 1 ครั้ง, ครั้งล่าสุดโดยคุณ อัจฉริยะ
ตอบพร้อมอ้างอิงข้อความนี้
  #3  
Old 26 กันยายน 2016, 15:20
nooonuii nooonuii ไม่อยู่ในระบบ
ผู้พิทักษ์กฎทั่วไป
 
วันที่สมัครสมาชิก: 25 พฤษภาคม 2001
ข้อความ: 6,408
nooonuii is on a distinguished road
Default

อ่านจบแล้วแบบว่าเอิ่ม..... สิ่งที่รอคอยมา 2-3 เดือนกระจ่างเสียที
__________________
site:mathcenter.net คำค้น
ตอบพร้อมอ้างอิงข้อความนี้
  #4  
Old 26 กันยายน 2016, 17:37
อัจฉริยะ อัจฉริยะ ไม่อยู่ในระบบ
กระบี่ไว
 
วันที่สมัครสมาชิก: 09 มิถุนายน 2016
ข้อความ: 217
อัจฉริยะ is on a distinguished road
Default

ผมรีบพิมพ์นะครับ บางทีอาจจะอ่านไม่รู้เรื่อง

ถ้าผิดพลาดตรงไหนก็ชี้แนะด้วย ถ้ามีเวลาจะ Edit ใหม่ ให้อ่านเข้าใจมากขึ้น

แต่ผมลองมาหลายวิธีละ ถ้าอันนี้ผิดอีก ก็ยอมแพ้ FLT แล้วครับ เหนื่อย
__________________
วิศวกรรมศาสตร์ พระจอมเกล้าลาดกระบัง รุ่น 38

26 กันยายน 2016 17:40 : ข้อความนี้ถูกแก้ไขแล้ว 1 ครั้ง, ครั้งล่าสุดโดยคุณ อัจฉริยะ
ตอบพร้อมอ้างอิงข้อความนี้
  #5  
Old 26 กันยายน 2016, 17:51
อัจฉริยะ อัจฉริยะ ไม่อยู่ในระบบ
กระบี่ไว
 
วันที่สมัครสมาชิก: 09 มิถุนายน 2016
ข้อความ: 217
อัจฉริยะ is on a distinguished road
Default

ตรงบรรทัดที่บอกว่า ((3)((5)^(k-1)))^(1/k) เป็นจำนวนอตรรกยะทุกค่าของ k ผมพิมพ์ผิดนะครับ

ต้องบอกว่าที่ k > 1 นะครับ เดี๋ยวแก้ไขใหม่
__________________
วิศวกรรมศาสตร์ พระจอมเกล้าลาดกระบัง รุ่น 38
ตอบพร้อมอ้างอิงข้อความนี้
  #6  
Old 26 กันยายน 2016, 19:11
อัจฉริยะ อัจฉริยะ ไม่อยู่ในระบบ
กระบี่ไว
 
วันที่สมัครสมาชิก: 09 มิถุนายน 2016
ข้อความ: 217
อัจฉริยะ is on a distinguished road
Default

ผมลืมใส่บทพิสูจน์เล็กๆ เอาไว้ด้วยครับ

จากข้อคาดการณ์ของผมนะ ยังไม่ได้พิสูจน์

====> รากที่ n ของจำนวนเต็มบวกใดๆ เป็นไปได้แค่ 2 อย่างคือ ''จำนวนเต็ม'' กับ ''จำนวนอตรรกยะ'' เท่านั้น

ถ้าข้อคาดการณ์นี้เป็นจริง บทพิสูจน์ผมจะถูกต้องครับ แต่ถ้าผิด บทพิสูจน์ผมก็จะผิดไปด้วย


เดี๋ยวผมจะกลับมาใหม่ พอดีงานเยอะ ใครมีอะไรชี้แนะก็บอกได้เลยครับผม

ต่อไปเตรียมพบกับ Goldbach's Conjecture proof ได้เลยครับ
__________________
วิศวกรรมศาสตร์ พระจอมเกล้าลาดกระบัง รุ่น 38

26 กันยายน 2016 19:12 : ข้อความนี้ถูกแก้ไขแล้ว 2 ครั้ง, ครั้งล่าสุดโดยคุณ อัจฉริยะ
ตอบพร้อมอ้างอิงข้อความนี้
  #7  
Old 26 กันยายน 2016, 20:12
อัจฉริยะ อัจฉริยะ ไม่อยู่ในระบบ
กระบี่ไว
 
วันที่สมัครสมาชิก: 09 มิถุนายน 2016
ข้อความ: 217
อัจฉริยะ is on a distinguished road
Default

อ้าว เกิดอะไรขึ้น ทำไมเข้าเว็ป vixra ไม่ได้ ผมกะว่าจะไป Edit ซะหน่อย คราวนี้สมบูรณ์แบบ ชัวร์ๆ

ใครเข้าได้บ้างครับ
__________________
วิศวกรรมศาสตร์ พระจอมเกล้าลาดกระบัง รุ่น 38
ตอบพร้อมอ้างอิงข้อความนี้
  #8  
Old 26 กันยายน 2016, 20:36
กขฃคฅฆง's Avatar
กขฃคฅฆง กขฃคฅฆง ไม่อยู่ในระบบ
บัณฑิตฟ้า
 
วันที่สมัครสมาชิก: 21 เมษายน 2015
ข้อความ: 419
กขฃคฅฆง is on a distinguished road
Default

ผมเข้าได้นะครับ
__________________
เหนือฟ้ายังมีอวกาศ
ตอบพร้อมอ้างอิงข้อความนี้
  #9  
Old 26 กันยายน 2016, 20:57
กขฃคฅฆง's Avatar
กขฃคฅฆง กขฃคฅฆง ไม่อยู่ในระบบ
บัณฑิตฟ้า
 
วันที่สมัครสมาชิก: 21 เมษายน 2015
ข้อความ: 419
กขฃคฅฆง is on a distinguished road
Default

ตรงข้อคาดการณ์ ถ้าไม่นับรากที่ไม่ใช่จำนวนจริงก็จริงครับ

__________________
เหนือฟ้ายังมีอวกาศ
ตอบพร้อมอ้างอิงข้อความนี้
  #10  
Old 26 กันยายน 2016, 21:15
อัจฉริยะ อัจฉริยะ ไม่อยู่ในระบบ
กระบี่ไว
 
วันที่สมัครสมาชิก: 09 มิถุนายน 2016
ข้อความ: 217
อัจฉริยะ is on a distinguished road
Default

อ้างอิง:
ข้อความเดิมเขียนโดยคุณ กขฃคฅฆง View Post
ตรงข้อคาดการณ์ ถ้าไม่นับรากที่ไม่ใช่จำนวนจริงก็จริงครับ

โอ้ว ขอบคุณมากครับ ปิดช่องโหว่บทพิสูจน์ผมพอดี ปริศนา 300 กว่าปี คลี่คลายแล้ว

ถ้างั้นบทพิสูจน์ผมก็ถูกแล้วล่ะครับ ขอบคุณมากๆครับ ผมก็เพิ่งเคยเจอทฤษฏีที่คุณยกมาเหมือนกัน

พอดีไม่ได้เรียนมาทางคณิตโดยตรง

ขอบคุณอีกครั้งครับ
__________________
วิศวกรรมศาสตร์ พระจอมเกล้าลาดกระบัง รุ่น 38

26 กันยายน 2016 21:16 : ข้อความนี้ถูกแก้ไขแล้ว 1 ครั้ง, ครั้งล่าสุดโดยคุณ อัจฉริยะ
ตอบพร้อมอ้างอิงข้อความนี้
  #11  
Old 26 กันยายน 2016, 21:23
อัจฉริยะ อัจฉริยะ ไม่อยู่ในระบบ
กระบี่ไว
 
วันที่สมัครสมาชิก: 09 มิถุนายน 2016
ข้อความ: 217
อัจฉริยะ is on a distinguished road
Default

ใครมีคำถามอะไรเกี่ยวกับ บทพิสูจน์ผมมั้ยครับ ? ผมจะตอบให้ เพราะทำเองกับมือ ไม่ได้เลียนแบบใคร
__________________
วิศวกรรมศาสตร์ พระจอมเกล้าลาดกระบัง รุ่น 38
ตอบพร้อมอ้างอิงข้อความนี้
  #12  
Old 26 กันยายน 2016, 21:23
Thgx0312555's Avatar
Thgx0312555 Thgx0312555 ไม่อยู่ในระบบ
กระบี่ประสานใจ
 
วันที่สมัครสมาชิก: 12 สิงหาคม 2011
ข้อความ: 885
Thgx0312555 is on a distinguished road
Default

คุณน่าจะรู้ตัวอยู่แล้วนะว่า proof ของคุณมันผิด

ตอนแรกที่คุณพิสูจน์โดยใช้กราฟ proof มันยังมีโอกาสถูกนะ
เพราะมันเป็นไอเดียใหม่ อาจจะเป็น proof ของ Fermat ก็ยังได้ด้วยซ้ำ

แต่ตอนที่คุณผิดครั้งนึงแล้ว คุณพยายามลนหา proof ใหม่ แถมยังเป็น proof ที่สั้นกว่าเดิมมากๆ
ไอเดียที่คุณคิดมันไม่มีทาง original แล้วล่ะ (เข้าใจว่าไอเดียที่ใช้น่าจะเปลี่ยนใหม่นะ)

ยิ่งที่ proof สุดท้ายมันน่าจะสั้นกว่า Fermat พิสูจน์แค่เคส $n=4$ แล้วด้วยซ้ำมันก็ไม่น่าใช่แล้วแหละ

แต่ลองเขียนมาเต็มๆก็ได้ ถ้าแนบภาษาไทยมาด้วยก็ดีเพราะว่าตอนนี้อ่านค่อนข้างยาก ก็จะได้มาคุยกันเลยว่าผิดตรงไหน

(หลังจากรอมาหลายเดือน...)
__________________
----/---~Alice~ จงรับรู้ไว้ ชื่อแห่งสีสันหนึ่งเดียวที่แสดงผล
---/---- ~Blue~ นี่คือ สีแห่งความหลังอันกว้างใหญ่ของเว็บบอร์ดนี้
ตอบพร้อมอ้างอิงข้อความนี้
  #13  
Old 26 กันยายน 2016, 21:31
อัจฉริยะ อัจฉริยะ ไม่อยู่ในระบบ
กระบี่ไว
 
วันที่สมัครสมาชิก: 09 มิถุนายน 2016
ข้อความ: 217
อัจฉริยะ is on a distinguished road
Default

อ้างอิง:
ข้อความเดิมเขียนโดยคุณ Thgx0312555 View Post
คุณน่าจะรู้ตัวอยู่แล้วนะว่า proof ของคุณมันผิด

ตอนแรกที่คุณพิสูจน์โดยใช้กราฟ proof มันยังมีโอกาสถูกนะ
เพราะมันเป็นไอเดียใหม่ อาจจะเป็น proof ของ Fermat ก็ยังได้ด้วยซ้ำ

แต่ตอนที่คุณผิดครั้งนึงแล้ว คุณพยายามลนหา proof ใหม่ แถมยังเป็น proof ที่สั้นกว่าเดิมมากๆ
ไอเดียที่คุณคิดมันไม่มีทาง original แล้วล่ะ (เข้าใจว่าไอเดียที่ใช้น่าจะเปลี่ยนใหม่นะ)

ยิ่งที่ proof สุดท้ายมันน่าจะสั้นกว่า Fermat พิสูจน์แค่เคส $n=4$ แล้วด้วยซ้ำมันก็ไม่น่าใช่แล้วแหละ

แต่ลองเขียนมาเต็มๆก็ได้ ถ้าแนบภาษาไทยมาด้วยก็ดีเพราะว่าตอนนี้อ่านค่อนข้างยาก ก็จะได้มาคุยกันเลยว่าผิดตรงไหน

(หลังจากรอมาหลายเดือน...)

ผมคิดแบบใช้กราฟแล้ว มันพิสูจน์ได้ไม่หมดทุกกรณีครับ

แต่ก็ยังได้อะไรติดไม้ติดมือมาบ้าง อย่างเช่น

จาก a^n + b ^n = c^n ที่ค่า n > ln(2)/ln(c /c-1) จะไม่มีค่า a,b,c เป็นจำนวนเต็มบวกพร้อมกันแน่นอน

อันนี้คือสิ่งที่ผมได้จากการวาดกราฟครับ แต่มันพิสูจน์ไม่ได้สำหรับ n < ln(2)/ln(c /c-1)


ผมเลยต้องล้มเลิกไอเดียนี้ไป ส่วนบทพิสูจน์นี้เป็นแนวคิดใหม่ครับ เดี๋ยวผมจะอธิบายให้ฟังอย่างละเอียดนะครับ
__________________
วิศวกรรมศาสตร์ พระจอมเกล้าลาดกระบัง รุ่น 38

26 กันยายน 2016 21:46 : ข้อความนี้ถูกแก้ไขแล้ว 6 ครั้ง, ครั้งล่าสุดโดยคุณ อัจฉริยะ
ตอบพร้อมอ้างอิงข้อความนี้
  #14  
Old 26 กันยายน 2016, 21:37
กขฃคฅฆง's Avatar
กขฃคฅฆง กขฃคฅฆง ไม่อยู่ในระบบ
บัณฑิตฟ้า
 
วันที่สมัครสมาชิก: 21 เมษายน 2015
ข้อความ: 419
กขฃคฅฆง is on a distinguished road
Default

บอกตามตรงครับว่าอ่านไม่เข้าใจ 55555

$(((3)(5)^{k-1})^{1/k})^d + (((4)(5)^{k-1})^{1/k})^d = 5^d$ นี่มายังไงหรอครับ d,k คืออะไร
__________________
เหนือฟ้ายังมีอวกาศ
ตอบพร้อมอ้างอิงข้อความนี้
  #15  
Old 26 กันยายน 2016, 21:42
อัจฉริยะ อัจฉริยะ ไม่อยู่ในระบบ
กระบี่ไว
 
วันที่สมัครสมาชิก: 09 มิถุนายน 2016
ข้อความ: 217
อัจฉริยะ is on a distinguished road
Default

อ้างอิง:
ข้อความเดิมเขียนโดยคุณ กขฃคฅฆง View Post
บอกตามตรงครับว่าอ่านไม่เข้าใจ 55555

$(((3)(5)^{k-1})^{1/k})^d + (((4)(5)^{k-1})^{1/k})^d = 5^d$ นี่มายังไงหรอครับ d,k คืออะไร
มาแล้วคำถาม

จริงๆสมการนี้ก็คือ สมการ 3^2 + 4^2 = 5^2 แหละครับ เหมือนกันเด๊ะๆ

เพียงแต่ผมแปลงรูปใหม่ ให้มันอยู่ในรูป ยกกำลัง d ซึ่ง d จะเป็นจำนวนเต็มใดๆก็ได้ตามใจชอบ


ส่วน ผมให้ d = kn ผมกำหนดให้มันไปเลยว่าให้ n = 2 , k = d/2

ลองแตกสมการออกมาสิครับ มันก็คือ 3^2 + 4^2 = 5^2 นี่เอง
__________________
วิศวกรรมศาสตร์ พระจอมเกล้าลาดกระบัง รุ่น 38
ตอบพร้อมอ้างอิงข้อความนี้
ตั้งหัวข้อใหม่ Reply


หัวข้อคล้ายคลึงกัน
หัวข้อ ผู้ตั้งหัวข้อ ห้อง คำตอบ ข้อความล่าสุด
Proof of the Fermat Last Theorem Phudis ปัญหาคณิตศาสตร์ทั่วไป 2 30 มกราคม 2014 20:57
หรม.&fermat's number กระบี่ทะลวงด่าน ทฤษฎีจำนวน 8 26 เมษายน 2012 14:07
Fermat's little theorem Amankris ทฤษฎีจำนวน 1 28 กุมภาพันธ์ 2012 18:36
ช่วยหาคำตอบให้ด้วยค่ะ (Fermat's last theorem) polarbear ปัญหาคณิตศาสตร์ทั่วไป 11 16 พฤษภาคม 2008 02:11
นำโปรแกรม Dijkstra's shortest path algorithm มาฝากคับ rigor ซอฟต์แวร์คณิตศาสตร์ 3 02 เมษายน 2007 16:50


กฎการส่งข้อความ
คุณ ไม่สามารถ ตั้งหัวข้อใหม่ได้
คุณ ไม่สามารถ ตอบหัวข้อได้
คุณ ไม่สามารถ แนบไฟล์และเอกสารได้
คุณ ไม่สามารถ แก้ไขข้อความของคุณเองได้

vB code is On
Smilies are On
[IMG] code is On
HTML code is Off
ทางลัดสู่ห้อง


เวลาที่แสดงทั้งหมด เป็นเวลาที่ประเทศไทย (GMT +7) ขณะนี้เป็นเวลา 20:38


Powered by vBulletin® Copyright ©2000 - 2024, Jelsoft Enterprises Ltd.
Modified by Jetsada Karnpracha